LSAT and Law School Admissions Forum

Get expert LSAT preparation and law school admissions advice from PowerScore Test Preparation.

 Administrator
PowerScore Staff
  • PowerScore Staff
  • Posts: 8916
  • Joined: Feb 02, 2011
|
#47091
Complete Question Explanation
(The complete setup for this game can be found here: lsat/viewtopic.php?t=11700)

The correct answer choice is (B)

The question stem eliminates two services—G and S—from being scheduled for the sixth day. As shown on the final diagram for the game, only G, L, and S can be scheduled for the sixth day, so when G and S are removed from consideration, we can conclude that L must be scheduled for the sixth day. Given that this is a Must Be True question, you should immediately seek this statement among the answer choices. Answer choice (B) states that inference, and answer choice (B) is correct.
Answer choice (A): This answer choice could be true, but it does not have to be true, and so it is incorrect. Note how question #4, answer choice (A) can be used to disprove this answer choice (A is actually the only answer from question #4 that can be used because neither G nor S is scheduled for the sixth day).

Answer choice (B): This is the correct answer choice.

Answer choice (C): This answer choice could be true, but it does not have to be true, and so it is incorrect.

Answer choice (D): This answer choice cannot be true (there would not be enough room to schedule both G and S), and so it is incorrect.

Answer choice (E): This answer choice could be true, but it does not have to be true, and so it is incorrect.
 tinglipmcd
  • Posts: 2
  • Joined: Feb 06, 2019
|
#62608
hi,
I have a quick question regarding #5:

the conclusion I drew from both the question stem and #5 stem was either gas, satellite or telephone can not be scheduled on the second, third or the sixth days,meaning G,S or T has to be scheduled on the first, fourth and the fifth day. Since both G and S have to be scheduled before after P, I was expecting something like " The telephone appointment is scheduled for the first day". Would this be correct if it appears in the answer choice?

Thanks!
 James Finch
PowerScore Staff
  • PowerScore Staff
  • Posts: 943
  • Joined: Sep 06, 2017
|
#62625
Hi tinglipmcd,

It looks like you're using sound reasoning for your Prephrase here, but just to make sure it's clear I'm going to explain my process for answering this question.

Based on the inferences we should have from the initial setup and the information given in this local question, two of the variables will have fixed positions (T in day 1 and L in day 6) and two other pairs of variables will be spread across two days, with either order being acceptable (P and W across days 2 and 3, in either order, and G and S in days 4 and 5, in either order).

With that information, as well the question stem asking what must be true, not what could be true, we should be looking for answer choices that involve the two fixed-position variables (T and L). Both are represented here, with (B) correctly noting that L must be on day 6 and (D) incorrectly claiming that T must go on day 4. By using the exact scope that the question stem is looking for, we can quickly narrow our answer choices down to two possible contenders and choose the correct one.

Hope this helps!

Get the most out of your LSAT Prep Plus subscription.

Analyze and track your performance with our Testing and Analytics Package.